Math, asked by varunvedhu608002, 9 hours ago

if the point P (-3,2/3)lies in the line segment joining the points A (- 5 ,-4) and B (- 2,3) then. pls help me ​

Answers

Answered by MaheswariS
1

\underline{\textbf{Given:}}

\mathsf{\left(-3,\dfrac{2}{3}\right)\;lies\;in\;the\;line\;joining\;the}

\textsf{Points A(-5,-4) and B(-2,3)}

\underline{\textbf{To find:}}

\textsf{The ratio in which P divides line segment AB}

\underline{\textbf{Solution:}}

\textsf{Let P divides line segment AB internally in the ratio m:n}

\textsf{By section formula,}

\mathsf{\left(\dfrac{m\,x_2+n\,x_1}{m+n},\dfrac{m\,y_2+n\,y_1}{m+n}\right)=\left(-3,\dfrac{2}{3}\right)}

\mathsf{\left(\dfrac{m(-2)+n(-5)}{m+n},\dfrac{m(3)+n(-4)}{m+n}\right)=\left(-3,\dfrac{2}{3}\right)}

\mathsf{\left(\dfrac{-2m-5n}{m+n},\dfrac{3m-4n}{m+n}\right)=\left(-3,\dfrac{2}{3}\right)}

\textsf{Equating corresponding co-ordinates on bothsides,}

\mathsf{\dfrac{-2m-5n}{m+n}=-3}

\mathsf{-2m-5n=-3(m+n)}

\mathsf{-2m-5n=-3m-3n}

\mathsf{-2m+3m=5n-3n}

\mathsf{m=2\,n}

\mathsf{\dfrac{m}{n}=\dfrac{2}{1}}

\implies\boxed{\mathsf{m:n=2:1}}

\therefore\textbf{P divides AB internally in the ratio 2:1}

\underline{\textbf{Find more:}}

In what ratio does the x- axis divide the line segment joining A(5,6) and B (2,-8)?​  

https://brainly.in/question/13579888

The point Q divides segment joining A (3, 5) and B (7, 9) in the ratio 2 : 3. Find the X-coordinate of Q.

https://brainly.in/question/38318873  

Similar questions